Communication systems and entanglement

In summary, those who are against superluminal communication often choose to look at a single set of 'entangled' particles as an example of communication between A(lice) and B(ob). For example, Patrick Van Esch: 'EPR, density matrices, and FTL signaling' that sees the two particles as ‘clearly defined subsystems’ - which seems surprising if they shares a common superposition. However, if we take a more communicative approach and look at a situation where the particles are measured over some period of time and based on agreements between the parties, superluminal communication is possible.
  • #36
This is only position '0'.

The transmitter polarizes p-photons horizontally or vertically.

Receiver PBS sends the now horizontally polarized s-photons at a double slit and vertically polarized against another.

As with Walburn should cause interference in both places.
 
Physics news on Phys.org
  • #37
Additional explanation:

Instead of letting a coincidens counter split the set into two subsets splits (Receiver)-PBS in the same two subsets. Therefore, coincidens couter spared.
 
  • #38
UChr said:
This is only position '0'.

The transmitter polarizes p-photons horizontally or vertically.

Receiver PBS sends the now horizontally polarized s-photons at a double slit and vertically polarized against another.

As with Walburn should cause interference in both places.

No ... in the Walborn DCQE experiment, there is no interference pattern shown in the single-photon measurements for the s-photon when the QWP's are in place. The interference pattern can only be recovered by looking at the coincidence counts between the s- and p-photon measurements when there is a properly oriented polarizer in the p-photon beam.
 
  • #39
I've apparently just confused by referring to Walburn.

In my gedanken experiment, there is no coincidence counter or QWP's.

Position 0 would cause interference:

The transmitter polarizes p-photons horizontally or vertically.

Receiver PBS sends the now horizontally polarized s-photons at a double slit and vertically polarized against another.

Position 1 should then erase this by changing the transmitter - comes later - if 0 seem ok.
 
  • #40
UChr said:
I've apparently just confused by referring to Walburn.

In my gedanken experiment, there is no coincidence counter or QWP's.

Position 0 would cause interference:

The transmitter polarizes p-photons horizontally or vertically.

Receiver PBS sends the now horizontally polarized s-photons at a double slit and vertically polarized against another.

Position 1 should then erase this by changing the transmitter - comes later - if 0 seem ok.

You still seem to be missing an important point .. the fringes and anti-fringes are features of the coincidence counts ONLY, which arise from setting the p-branch polarizer to the angle of the QWP over EITHER the left-slit (45º, for fringes), or the right slit (135º, for anti-fringes). So you need both the QWP's and the coincidence counters to observe them.
 
  • #41
None QWP’s:

One DS receives exclusively horizontally polarized photons - this should lead to interference - and the other DS receives only vertically - and again interference (horisontally - vertically = half wave = fringes and anti-fringes).

Walburn Re: This is the type of interference the QWP s later erase.
 
  • #42
UChr said:
None QWP’s:

One DS receives exclusively horizontally polarized photons - this should lead to interference - and the other DS receives only vertically - and again interference (horisontally - vertically = half wave = fringes and anti-fringes).

Walburn Re: This is the type of interference the QWP s later erase.

No No No No .. read my posts again .. fringes and anti-fringes ONLY arise due to BOTH the QWP's AND the coincidence counting .. your setup has neither. Also, read the recent threads in this forum on DCQE .. that may help you understand why what you are proposing won't work.
 
  • #43
Yes and no. It is my assumption that when horizontally gives fringes - vertically should give (because of reflection - half-wave) anti-fringes.

Walburn re:

If you look at arXiv – text, I think about interference shown on figure 2.

Grad.physics text, p 6 schedule: ‘Detected polarization for photon p’ : x / y. (my horizontally / vertically).

American Scientist: p 340 Volumen 91. ‘Now imaging that we repeat Yong’s experiment with many horizontally polarized photons. Behind the slits we insert two QWP […] Remarkably, the interference fringes will disappear.’
 
  • #44
UChr said:
Yes and no. It is my assumption that when horizontally gives fringes - vertically should give (because of reflection - half-wave) anti-fringes.

You need to define what you mean by fringes and anti-fringes, because it is necessarily something different than in the DCQE experiments. In your setup, you have two different double-slits (without QWP's) with two different detecting screens. Each apparatus does a single-photon double-slit experiment .. why are you expecting to see something different at the detection screens, and what specifically is the difference that you are expecting to see?

Also, I guess you realize that you need to send many many photons to observe an interference pattern, right? You haven't mentioned that explicitly, and from some of your posts I get the impression you think you will see "interference patterns" as discrete events.
 
  • #45
SpectraCat said:
You need to define what you mean by fringes and anti-fringes, because it is necessarily something different than in the DCQE experiments. In your setup, you have two different double-slits (without QWP's) with two different detecting screens. Each apparatus does a single-photon double-slit experiment .. why are you expecting to see something different at the detection screens, and what specifically is the difference that you are expecting to see?

Also, I guess you realize that you need to send many many photons to observe an interference pattern, right? [...]

OK – fringes both places.

Yes - many - many. (Maybe many is enough. I will believe that it is enough to measure 4 to 5 selected locations at the detection screen to observe the difference between constructive and destructive interference or not - but it's technology - so let it just be many - many.).
 
  • #46
Summary:

T(0) = Transmitter position 0:
A PBS(0) which polarizes p-photons horizontally or vertically.

Receiver:
A PBS(0) which sends the now horizontally polarized s-photons at a double slit and vertically polarized against another.

After a while, the many many photons forming interference pattern on both detector screens.
---
New: T(1) = Transmitter position 1:

A PBS(45) which polarizes p-photons 45 or -45 degrees to the horizontal = diagonally right or left / diagonally positive or negative.

Effect on the s-photons:
a) The s-photons will be polarized diagonally negative or positive.
b) Because of the difference between transmission and reflection: a half wave difference between diagonally negative and positive.

When s-photons meetings the receiver PBS(0) they will have a fifty - fifty chance of being polarized horizontally or vertically:
ie the beam against both double slits will consist of a mix of photons coming from diagonally negative or positive = with a half wave difference.

One must therefore expect no / very little interference in this case.

(It should be possible to do something similar with a source of circularly polarized entangled photons. - could possibly be discussed later) 
 
  • #47
UChr said:
Summary:

T(0) = Transmitter position 0:
A PBS(0) which polarizes p-photons horizontally or vertically.

Receiver:
A PBS(0) which sends the now horizontally polarized s-photons at a double slit and vertically polarized against another.

After a while, the many many photons forming interference pattern on both detector screens.
---
New: T(1) = Transmitter position 1:

A PBS(45) which polarizes p-photons 45 or -45 degrees to the horizontal = diagonally right or left / diagonally positive or negative.

Effect on the s-photons:
a) The s-photons will be polarized diagonally negative or positive.
b) Because of the difference between transmission and reflection: a half wave difference between diagonally negative and positive.

When s-photons meetings the receiver PBS(0) they will have a fifty - fifty chance of being polarized horizontally or vertically:
ie the beam against both double slits will consist of a mix of photons coming from diagonally negative or positive = with a half wave difference.

One must therefore expect no / very little interference in this case.

(It should be possible to do something similar with a source of circularly polarized entangled photons. - could possibly be discussed later) 

In neither case will there be any interference. Entangled photons do NOT exhibit self interference UNLESS which path information is ERASED. This cannot be made to happen at will.
 
  • #48
DrChinese said:
In neither case will there be any interference. Entangled photons do NOT exhibit self interference UNLESS which path information is ERASED. This cannot be made to happen at will.

'which path information' - no QWP so where?
 
  • #49
UChr said:
'which path information' - no QWP so where?

What does the quarter wave plate have to do with anything? Or allow me to rephrase: a QWP does not erase anything. A pair of them "might" but only with coincidence counting and without the beam splitter. But the setup you described won't give any interference.
 
  • #50
? # 47: ' UNLESS which path information is ERASED.´
There is no which path information to erase.

DrChinese said:
But the setup you described won't give any interference.

Could you please write why T(0) should not work? - (because T(1) is not intended to give interference).
 
  • #51
UChr said:
? # 47: ' UNLESS which path information is ERASED.´
There is no which path information to erase.

Ah, but there is! You just don't plan to look at it.

See Zeilinger, page 290, figure 2, there is no direct interference pattern for entangled photons:

Experiment and the foundations of quantum physics (1999)
 
  • #52
By the way, I fell victim to this little twist when I was first looking at this setup. I was sure I had found a way to signal FTL... but nature is tricky!
 
  • #53
DrChinese said:
By the way, I fell victim to this little twist when I was first looking at this setup. I was sure I had found a way to signal FTL... but nature is tricky!

Nice to hear. Yes very tricky.

Re 290, Figure 2: I have some questions - that might help me further:

To remove the suspicion of Which Path information I could try to modify my experimental set-up:
I could use optical fibers to transport p - photons from the source until the transmitter's PBS and also from source until the receiver’s PBS, and I could replace both the Double Slits with DS - substitutes:
1: a half-silvered mirror followed by optical fibers - adjusted in length equal to the difference between transmitted and reflected - and compiled to emulate a DS

2: an interferometer.
Arises then similar problems - or should this solve the WP – problem?

Re Walborn:
Figure 2 is without QWP and without polarizer - but shows interference. How does this work?

With the two QWP’s in place: Should then just a laserbeam with diagonal (positive) polarized photons cause interference - or is the presence of entanglement decisive?
 
  • #54
UChr said:
Nice to hear. Yes very tricky.

Re 290, Figure 2: I have some questions - that might help me further:

To remove the suspicion of Which Path information I could try to modify my experimental set-up:
I could use optical fibers to transport p - photons from the source until the transmitter's PBS and also from source until the receiver’s PBS, and I could replace both the Double Slits with DS - substitutes: ...

To get interference for Alice, Bob must be detected as a wave and not a localized particle. Sending Bob through a fiber will turn Bob particle-like and that means no interference for Alice. And vice versa! Again, you must look at the entire context. It is not easy to detect both members of a pair as wave-like and even in principle this cannot be done on demand.

I didn't try to work through your examples all the way as these Quantum Eraser discussions get sidetracked almost immediately with the details of the setup and it gets impossible to move past that. The point is always the same as I mention: you don't get interference from photons in which it is possible, in principle, to know the which path information - regardless of whether you actually know it or not.
 
  • #55
DrChinese said:
To get interference for Alice, Bob must be detected as a wave and not a localized particle. Sending Bob through a fiber will turn Bob particle-like and that means no interference for Alice. And vice versa!

then no fiber - I thought maybe it was more smart - but it is a gedanken experiment, so no need to unnecessary practical difficulties.

So transmitter PBS(0) or PBS(45).
Receiver: a PBS(0) followed by two Mach-Zehnder interferometers - with for example a BS = half silvered mirror.

DrChinese said:
I didn't try to work through your examples all the way as these Quantum Eraser discussions get sidetracked almost immediately with the details of the setup and it gets impossible to move past that.

Maybe one of those who has studied the Walborn experiment, answer my two questions?

(Walborn figure 2 is without QWP and without polarizer - but shows interference. How does this work?

With the two QWP’s in place: Should then just a laserbeam with diagonal (positive) polarized photons cause interference - or is the presence of entanglement decisive?)
 
  • #56
Which Walborn experiment? Please link to the paper you are referencing.
 
  • #57
UChr said:
then no fiber - I thought maybe it was more smart - but it is a gedanken experiment, so no need to unnecessary practical difficulties.

So transmitter PBS(0) or PBS(45).
Receiver: a PBS(0) followed by two Mach-Zehnder interferometers - with for example a BS = half silvered mirror.



Maybe one of those who has studied the Walborn experiment, answer my two questions?

(Walborn figure 2 is without QWP and without polarizer - but shows interference. How does this work?

With the two QWP’s in place: Should then just a laserbeam with diagonal (positive) polarized photons cause interference - or is the presence of entanglement decisive?)

Have you looked at this?

http://grad.physics.sunysb.edu/~amarch/

http://grad.physics.sunysb.edu/~amarch/Walborn.pdf
 
Last edited by a moderator:
  • #58
SpectraCat said:
Which Walborn experiment? Please link to the paper you are referencing.

arXiv:quant-ph/0106078v1 13 juni 2001
 

Similar threads

  • Quantum Physics
Replies
2
Views
738
  • Quantum Physics
Replies
1
Views
1K
  • Quantum Physics
Replies
4
Views
951
  • Quantum Physics
Replies
4
Views
1K
Replies
1
Views
804
Replies
56
Views
5K
Replies
24
Views
1K
  • Quantum Physics
Replies
8
Views
687
Replies
11
Views
1K
Replies
9
Views
1K
Back
Top